Đến nội dung

dogsteven nội dung

Có 1000 mục bởi dogsteven (Tìm giới hạn từ 29-04-2020)



Sắp theo                Sắp xếp  

#689632 Z luôn thuộc $(J)$, là đường tròn bàng tiếp góc A của tam giác APQ

Đã gửi bởi dogsteven on 05-08-2017 - 16:09 trong Hình học

Bài này dùng hợp phép vị tự thôi bạn.




#538275 Xác định vị trí M;N để MN có giá trị nhỏ nhất

Đã gửi bởi dogsteven on 16-12-2014 - 21:08 trong Hình học

(a) Gọi $M'$ và $N'$ là hình chiếu của $M, N$ trên $BC$. $MN\geqslant M'N'=\dfrac{BC}{2}$

(b) $AB=AC=a, AM=x \Rightarrow AN = a-x$

$$S_{AMN}=\dfrac{x(a-x)\sin A}{2} \leqslant \dfrac{a^2\sin A}{8} \Rightarrow S_{AMN}=\dfrac{a^2\sin A}{8} \Leftrightarrow AM=AN$$




#554833 Xác định vị trí của A để MN có độ dài ngắn nhất

Đã gửi bởi dogsteven on 18-04-2015 - 15:59 trong Hình học

$O_1O_2CB$ là hình bình hành $\to$ $O_1O_2FE$ là hình bình thành $\to$ $AMHN$ nội tiếp $\to MN// BC\to$ soạn đề cương môn văn -_-




#544024 xy+yz+zx≥x+y+z

Đã gửi bởi dogsteven on 13-02-2015 - 20:09 trong Bất đẳng thức - Cực trị

sao bạn có thể nghĩ ra cách đổi biến thỏa mãn điều kiện bài toán hay như vậy?

Chú ý đẳng thức sau: $xy(x+y)+yz(y+z)+zx(z+x)+2xyz=(x+y)(y+z)(z+x)$. Khi đó $\dfrac{2xy(x+y)+2yz(y+z)+2zx(z+x)+8xyz}{(x+y)(y+z)(z+x)}=4$

Từ đó ta có phép đặt trên.




#543999 xy+yz+zx≥x+y+z

Đã gửi bởi dogsteven on 13-02-2015 - 18:35 trong Bất đẳng thức - Cực trị

Đặt $x=\dfrac{1}{a}, y=\dfrac{1}{b}$ và $z=\dfrac{1}{c}$ thì ta có $ab+bc+ca+abc=4$ hay có thể đặt $a=\dfrac{2a}{b+c}, b=\dfrac{2b}{c+a}, c=\dfrac{2c}{a+b}$

Bất đẳng thức trở thành: $a(a+b)(a+c)+b(b+c)(b+a)+c(c+a)(c+b)\geqslant 2ab(a+b)+2bc(b+c)+2ca(c+a)$

$\Leftrightarrow a^3+b^3+c^3+3abc\geqslant ab(a+b)+bc(b+c)+ca(c+a) \\ \Leftrightarrow (a+b-c)(a-b)^2+c(b-c)(a-c)\geqslant 0$

Giả sử $c=\text{min}\{a,b,c\}$ thì bất đẳng thức đúng.




#585113 xP(x)=(x-10)P(x-1)

Đã gửi bởi dogsteven on 26-08-2015 - 19:15 trong Đa thức

Cho $x=10\Rightarow f(10)=0$

Cho $x=11\Rightarrow (11)=0$

Cho $x=12\Rightarrow f(12)=0$

...

P(x) vô số nghiệm nên $P(x)\equiv 0$




#540070 VMO 2015

Đã gửi bởi dogsteven on 09-01-2015 - 11:14 trong Thi HSG Quốc gia và Quốc tế

Câu 2: $3(a^2+b^2+c^2)\geqslant (a+b+c)(\sqrt{ab}+\sqrt{bc}+\sqrt{ca})+(a-b)^2+(b-c)^2+(c-a)^2 \Leftrightarrow \sum (\sqrt{b}-\sqrt{c})^2 \geqslant 0$

$ (a+b+c)(\sqrt{ab}+\sqrt{bc}+\sqrt{ca})+(a-b)^2+(b-c)^2+(c-a)^2 \geqslant (a+b+c)^2\Leftrightarrow a^2+b^2+c^2+(a+b+c)(\sqrt{ab}+\sqrt{bc}+\sqrt{ca})-2(ab+bc+ca)\geqslant 2(ab+bc+ca)$

Áp dụng bất đẳng thức AM-GM: $ (a+b+c)(\sqrt{ab}+\sqrt{bc}+\sqrt{ca})-2(ab+bc+ca)\geqslant (a+b+c)\left(\dfrac{2bc}{b+c}+\dfrac{2ca}{c+a}+\dfrac{2ab}{a+b}\right)-2(ab+bc+ca)=2abc.\sum \dfrac{1}{b+c}\geqslant \dfrac{9abc}{a+b+c}$

Do đó ta cần chứng minh: $a^2+b^2+c^2+\dfrac{9abc}{a+b+c}\geqslant 2(ab+bc+ca)\Leftrightarrow (a-b)[a(a-c)-b(b-c)]+c(a-c)(b-c)\geqslant 0$

Giả sử $a\geqslant b\geqslant c$ thì $(a-b)[a(a-c)-b(b-c)]+c(a-c)(b-c)\geqslant (a-b)^2(b-c)\geqslant 0$




#540128 VMO 2015

Đã gửi bởi dogsteven on 09-01-2015 - 19:22 trong Thi HSG Quốc gia và Quốc tế

Bài 1 :  

 

a.  Với $a=0$ thì dãy viết lại như sau :         $\left\{\begin{matrix} u_{1}=3 & \\ u_{n+1}=\frac{1}{2}u_{n}+\frac{1}{4}\sqrt{u^{2}_{n}+3} & \end{matrix}\right.$

 

     Th1:   $$u_{n}\in \begin{bmatrix} 1;\infty \end{bmatrix}$$  (**)

 

Xét  $u_{n+1}-u_{n}=\frac{1}{4}\sqrt{u^{2}_{n}+3}-\frac{u_{n}}{2}$ (*)

 

Giả sử $u_{n}$  là hàm tăng thì (*) $\Leftrightarrow \sqrt{u^{2}_{n}+3} >  2u_{n}\Leftrightarrow u_{n}<1$  ( vô lý ) 

 

nên $u_{n}$ là hàm giảm  mà kết hợp với (**) nên $u_{n}$ có giới hạn hữu hạn .   

 

Gọi    $lim u_{n}=L$   ,  chuyển qua giới hạn ta có :    $L=1$  nên    $lim u_{n}=1$

 

  Hiển nhiên ta có $u_{n}>0$ 

 

 Th2 :        $0< u_{n}\leq 1$ (***)  ,  tương tự như trên ta cũng chứng minh được  $u_{n}$ là hàm số tăng mà kết hợp với  (***)

 

ta được  $u_{n}$ tăng và bị chặn trên nên    $lim u_{n}=1$

 

b.         Th1 :   $u_{n}\in \begin{bmatrix} 1;\infty \end{bmatrix}$  ,  chứng minh tương tự câu a  nên dãy có giới hạn hữu hạn 

 

            Th2 :   $0 < u_{n}\leq 1 $  ta cũng sẽ chứng minh $u_{n}$ là hàm tăng như sau :      

 

Xét :  $u_{n+1}-u_{n}=\frac{n^{2}}{4n^{2}+a}\sqrt{u^{2}_{n}+3}-\frac{1}{2}u_{n}$

 

Sau đó sử dụng đánh giá  :   $a <1$  rồi đưa về biểu thức sau :   $u^{2}_{n}=\frac{12n^{4}}{12n^{4}+8n^{2}+1}<1 \rightarrow u_{n}<1$  (đúng )

 

nên $u_{n}$  tăng và bị chặn trên nên  $u_{n}$ có giới hạn hữu hạn 

Câu (a)

Chứng minh được $u_{2}<u_{1}$

Giả sử $u_{n+1}<u_{n}$ đúng với $n=k$

Với $n=k+1$ thì $u_{k+1}-u_{k+2}=\dfrac{u_{k}-u_{k+1}}{2}+\dfrac{\sqrt{u_{k}^2+3}-\sqrt{u_{k+1}^2+3}}{4}>0$ theo giả thiết quy nạp.

Vậy $u_{n}$ là dãy giảm.




#540068 VMO 2015

Đã gửi bởi dogsteven on 09-01-2015 - 11:08 trong Thi HSG Quốc gia và Quốc tế

Bài 4: (a) Câu này anh chép nhầm đề rồi. Phải là $\dfrac{BD}{CD}$.

Gọi đường cao $AA'$ và $S\equiv (I)\cap AB, R\equiv (I) \cap AC$

Khi đó $RS||BC \Rightarrow \dfrac{BD^2}{CD^2}=\dfrac{BF.BS}{CE.CR}=\dfrac{BF.BA}{CE.CA}=\dfrac{BA'}{CA'}=\dfrac{\cot B}{\cot C}$

(b) Xét trục đẳng phương của $(I), (FECB), (HBC), (TQP)$ thì tiếp tuyến tại $T$ của $(O)$, $EF, PQ$ và $BC$ đồng quy tại $U$

Khi đó để dàng chứng minh được $UT$ cũng là tiếp tuyến của $(TMN)$ và $(TM,TB)\equiv (TC,TN) \pmod{\pi}$ nên phân giác góc $NTM$ luôn đi qua điểm chính giữa cung $BC$ của (O) không chứa $A$




#643021 VMF's Marathon Hình học Olympic

Đã gửi bởi dogsteven on 30-06-2016 - 21:38 trong Hình học

Em có một bài toán tương tự khác.

Cho tam giác $ABC$ và điểm $P$ nằm trên trung trực $BC$, đường tròn ngoại tiếp tam giác $(PBC)$ cắt $AB, AC$ tại $F, E$. Gọi $Q$ là điểm đẳng giác của $P$ trong tam giác $ABC$. Khi đó $PQ, BE, CF$ đồng quy.




#644017 VMF's Marathon Hình học Olympic

Đã gửi bởi dogsteven on 07-07-2016 - 19:55 trong Hình học

Lời giải đầy đủ bài toán 67 có thể tham khảo link.

 

Theo đề nghị của anh Dogsteven mình xin đề xuất bài toán mới sau.

$\boxed{\text{Bài toán 68}}$ (AoPS) Cho tam giác $ABC$ với tâm đường tròn $Euler$ là $N.D,E,F$ là hình chiếu của $N$ lên $BC,CA,AB$. Chứng minh rằng đường thẳng $Euler$ của các tam giác $AEF,BFD,CDE,ABC$ đồng quy.

 

Bài này chính là một bài trong đề thi 30/4 lớp 10 năm nay. 




#642921 VMF's Marathon Hình học Olympic

Đã gửi bởi dogsteven on 30-06-2016 - 09:27 trong Hình học

Với bài 52, em xin đề xuất một cách giải dùng diện tích.

Gọi $Q$ là điểm đối xứng của $P$ qua $O$, gọi $X, Y, Z, T$ là hình chiếu của $Q$ trên $AB, BC, CD, DA$ ($O$ là tâm $(EFGH)$)

Ta có $PQXE$ là hình thang vuông, mà $O$ là trung điểm $PQ$ nên $OE=OX$ hay $X\in (O)$

Chứng minh tương tự ta suy ra $Y, Z, T\in (O)$, bằng việc biến đổi góc: $\widehat{DQT}=\widehat{DZT}=\widehat{DHG}=\widehat{DPG}$

Do đó $DP, DQ$ đẳng giác góc $D$ tứ giác $ABCD$, chứng minh tương tự với các góc còn lại.

Ta có: $\dfrac{AX}{AH}=\dfrac{AT}{AE}=\dfrac{QT}{PE}$ nên $AX.PE=AH.QT$, tương tự ta có $BX.PE=BF.QY$

Cộng vế theo vế ta suy ra $S_{PAB}=S_{QAH}+S_{QBF}$, một cách tương tự ta có: $S_{PCD}=S_{QDH}+S_{QCF}$

Do đó $S_{PAB}+S_{PBC}=S_{QAD}+S_{QBC}$ hay $S_{PBC}+S_{QBC}+S_{PAD}+S_{QAD}=S_{ABCD}$

Mà $2S_{OBC}=S_{PBC}+S_{QBC}$ và $2S_{OAD}=S_{PAD}+S_{QAD}$ nên $S_{OBC}+S_{OAD}=\dfrac{S_{ABCD}}{2}$

$BC$ cắt $AD$ tại $U$, dựng $V \in BC, W\in AD$ sao cho $UV=BC$ và $UW=AD$

Khi đó $S_{OVW}=S_{OVU}+S_{OWU}-S_{UVW}=S_{OBC}+S_{OAD}-S_{UVW}=\dfrac{S_{ABCD}}{2}-S_{VWU}$

Gọi $M$ là trung điểm $AC$ thì $S_{MVW}=S_{MVU}+S_{MWU}-S_{UVW}=S_{MBC}+S_{MAD}-S_{UVW}=\dfrac{S_{ABCD}}{2}-S_{VWU}$

Như vậy $OM || VW$, tương tự nếu $N$ là trung điểm $BD$ thì  $ON || VW$ hay $O, M, N$ thẳng hàng.




#642805 VMF's Marathon Hình học Olympic

Đã gửi bởi dogsteven on 29-06-2016 - 17:57 trong Hình học

$\boxed{\text{Bài toán 46}}$ (Theo đề nghị của anh halloffame)

$P$ là giao điểm của $(BAD)$ và $(CAE)$. $PE$ cát $AB$ tại $T$, $PD$ cắt $AC$ tại $Z$ thì $ZT$ là trục đẳng phương của $(O')$ và $(PDE)$

Khi đó $180^o-\widehat{AZP}=\widehat{DAC}+\widehat{ADZ}=\widehat{ABP}+\widehat{DAP}+\widehat{PAC}=\widehat{TBE}+\widehat{TED}=\widehat{ATP}$

Do đó tứ giác $ATPZ$ nội tiếp nên $\widehat{TZD}=\widehat{TAP}=\widehat{ZDE}$ hay $TZ || BC$

Do đó $O'$ thuộc trung trực $DE$ nên $OO'\perp BC$




#642767 VMF's Marathon Hình học Olympic

Đã gửi bởi dogsteven on 29-06-2016 - 13:25 trong Hình học

$\boxed{\text{Bài toán 49}}$

Ta có $DE.DB = DF.DC = DA.DP$ nên $D$ là tâm vị tự trong của $(AEF)$ và $(PBC)$

Gọi $R$ là tâm đường tròn ngoại tiếp tam giác $PBC$ thì $M, D, R$ thẳng hàng.

Gọi $X$ là tâm của $(AED)$ thì ta có $\widehat{MXD}=180^o-\widehat{EXD}-\widehat{LXE}=\widehat{AED}-\widehat{EAD}=\widehat{AED}-\widehat{PBD}=\widehat{AIP}=\widehat{MLR}$ với $I$ là giao điểm của $AE$ và $PB$
Do đó $DX || LR$, mà $XN || LK$ nên $\dfrac{MN}{MQ}=\dfrac{MX}{ML}=\dfrac{MD}{MR}$, do đó $RQ || ND \perp BC$ mà $R$ thuộc trung trực $BC$ nên $Q$ thuộc trung trực $BC$



#690193 VMF's Marathon Hình học Olympic

Đã gửi bởi dogsteven on 11-08-2017 - 02:16 trong Hình học

Hướng chứng minh cho Bài toán 196:

Chứng minh $J$ là tâm nội tiếp và $LP$ vuông góc với $QR$ => $LG$ đi qua trực tâm $AEF$ và $AQR$ nên $LG$ vuông góc với đường thẳng Gauss.

@halloffame: Lời giải này chưa hoàn thiện, đầy đủ.




#642856 VMF's Marathon Hình học Olympic

Đã gửi bởi dogsteven on 29-06-2016 - 21:03 trong Hình học

$\boxed{\text{Bài toán 51}}$ Em nghĩ đề là $BN, CM, EF$ đồng quy tại $R$

Gọi $R$ là giao điểm của $BN, CM$, khi đó ta có $B(EF,RC)=B(QF,ND)=Q(EF,ND)=Q(EF,MD)=C(EF,RB)$ nên $E,F,R$ thẳng hàng.

Ngoài ra $Q(RP,BC)=N(RQ,EF)=N(BQ,EF)=-1$, mà $QP$ là phân giác ngoài góc $BQC$ nên $QR \perp PQ$

Do đó $\widehat{RQA}=90^o-\widehat{AQP}=\widehat{PAO}$

$\boxed{\text{Bài toán 52}}$ Cho tứ giác $ABCD$ với điểm $P$ nằm trong nó thỏa mãn $\widehat{APB}+\widehat{CPD}=180^o$. Gọi $E, F, G, H$ là hình chiếu của $P$ trên $AB, BC, CD, DA$. Chứng minh rằng tâm đường tròn ngoại tiếp tứ giác $EFGH$ nằm trên đường thẳng đi qua trung điểm $AC$ và $BD$




#635173 VMF's Marathon Hình học Olympic

Đã gửi bởi dogsteven on 24-05-2016 - 12:43 trong Hình học

$\boxed{\text{Bài toán 1.}}$

Gọi $N$ là điểm chính giữa cung $BC$ không chứa $A$. Ta có $NI=NB=NO$ nên $\widehat{OIX}=\dfrac{180^o-\widehat{ONA}}{2}=80^o$

Ngoài ra $\widehat{OXI}=\widehat{OBA}+\dfrac{\widehat{BAC}}{2}=80^o$ nên $OI=OX$

Ta có $\widehat{OAI}=\widehat{OCE}$ và $\widehat{AOI}=\widehat{OIX}-\widehat{OAI}=60^o=\widehat{EOC}$ mà $OA=OC$ nên $\Delta OIA = \Delta OEC$

Do đó $OI=OE$. Mà $\widehat{OEK}=\widehat{OIX}=80^o$ $\widehat{KOE}=\widehat{ONA}=20^o$ nên $OK=OE$

Dễ thấy $\Delta ICA=\Delta EAC$ nên $\widehat{EAC}=20^o$, từ đó $\widehat{MAE}=10^o$ nên tứ giác $AEMB$ nội tiếp.

Do đó $\widehat{MEB}=\widehat{MAB}=30^o=\widehat{IBE}$ nên $EL || BI$

Gọi $T$ là giao của $BI$ với $AC$ thì $BTC$ cân tại $T$ và $O,E,T$ thẳng hàng.

Dễ thấy $EI|| CT$ nên $I,L$ đối xứng với nhau qua $OE$, suy ra $OI=OL$

Ngoài ra $\widehat{LEC}=50^o$ và $\widehat{KFL}=50^o$ nên $F\in (KEL)$

Ta có điều phải chứng minh.

P/s. Xin giải lại bài toán 1 với lời giải khác.

$$\begin{array}{| l | l |} \hline Ngockhanh99k48 & 1\\ \hline IHateMath & 1\\ \hline fatcat12345 & 1\\ \hline dogsteven & 1\\ \hline \end{array}$$




#642931 VMF's Marathon Hình học Olympic

Đã gửi bởi dogsteven on 30-06-2016 - 11:08 trong Hình học

$\boxed{\text{Bài toán 53.}}$ Với bài toán này, ta có $\widehat{EPF}+\widehat{BPC}=180^o$ nên có thể áp dụng bài toán trước để suy ra điều phải chứng minh.

Ngoài ra còn có một cách giải khác, sử dụng đến bổ đề ERIQ.

Vẫn tiếp tục ý tưởng cũ, gọi $X, Y, Z, T$ là hình chiếu của $P$ trên $FE, EC, CB, BF$, $O$ là tâm $(XYZT)$, $Q$ đối xứng với $P$ qua $O$, $U, V, W$ là hình chiếu của $Q$ trên $FE, EC,CB$

Chứng minh tương tự lời giải trên ta suy ra $U, V, W\in (XYZT)$ và $P, Q$ tạm gọi là đẳng giác trong tứ giác $ABCD$

Ta có $\widehat{EUV}=\widehat{EYX}=\widehat{EPX}=\widehat{EFP}$ nên $UV || FC$, tương tự ta có $VW || EB$ nên $UW$ là đường kính của $(XYZT)$ hay $O$ là trung điểm $UV$

Ta có $\widehat{CQW}=\widehat{CVW}=\widehat{CEP}=\widehat{QEU}$ nên $\Delta CQW \sim \Delta QEU$, do đó $\dfrac{CW}{WQ}=\dfrac{QU}{UE}$

Tương tự ta có $\dfrac{BW}{WQ}=\dfrac{QU}{UF}$, từ đó suy ra $\dfrac{CW}{BW}=\dfrac{UF}{UE}$

Áp dụng bổ đề ERIQ cho ta điều phải chứng minh.




#638988 VMF's Marathon Hình học Olympic

Đã gửi bởi dogsteven on 08-06-2016 - 19:33 trong Hình học

$\boxed{\text{Bài toán 24.}}$

Bổ đề 1. Cho tam giác $ABC$ với hai điểm liên hợp đẳng giác là $P, Q$. Gọi $K, L$ lần lược là trực tâm các tam giác $PAB, PAC$. Khi đó $KL$ đi qua hình chiếu $D$ của $Q$ trên $BC$. 

Chứng minh.

Gọi $E, F$ là hình chiếu của $P$ trên $AC, AB$. Khi đó:

$\Delta BPF \sim \Delta BQD \Rightarrow \dfrac{BD}{DQ}=\dfrac{BF}{FP}$

$\Delta CPE \sim \Delta CQD \Rightarrow \dfrac{CD}{DQ}=\dfrac{CE}{EP}$

$\Delta BKF \sim \Delta PAF \Rightarrow \dfrac{BK}{PA}=\dfrac{BF}{FP}$

$\Delta CLE \sim \Delta PAE \Rightarrow \dfrac{CL}{PA}=\dfrac{CE}{EP}$

Từ đó ta suy ra $\dfrac{BK}{CL}=\dfrac{CD}{DC}$ mà $BK || CL$ nên ta có điều phải chứng minh.

Bổ đề 2.  Cho tam giác $ABC$ với hai điểm liên hợp đẳng giác là $P, Q$. Gọi $K, L$ lần lược là trực tâm các tam giác $PAB, PAC$. Gọi $D$ là hình chiếu của $Q$ trên $BC$. Gọi $E, F$ là hình chiếu của $P$ trên $BC, AB$. Gọi $T$ là giao điểm của $KL$ và $(DEF)$ khác $D$. Khi đó $AT \perp KL$

Chứng minh.

Ta có $\widehat{FTD}=\widehat{FEB}=90^o-\widehat{QBE}=90^o-\widehat{PBA}=\widehat{FAK}$

Do đó tứ giác $AKTF$ nội tiếp nên $AT\perp KL$

Trở lại bài toán.

Gọi $E, F$ là hình chiếu của $P, Q$ trên $BC$. $E', F'$ là giao khác $E, F$ của $PE, QF$ với đường tròn Pedal tương ứng với $P$ của tam giác $ABC$.

Áp dụng bổ đề 1 ta suy ra $KL$ đi qua $E$ và $MN$ đi qua $F$

Gọi $T, S$ là giao của $KL, MN$ với $(E'EF)$ khác $E, F$, áp dụng bổ đề 2 ta suy ra $AE'$ đi qua $T$ và $AF'$ đi qua $S$

Ta có tứ giác $ARTS$ nội tiếp nên $\widehat{ARS}=\widehat{ATS}=\widehat{E'TS}=\widehat{PES}$

Do đó $AR || PE$ nên $AR \perp BC$

Một trường hợp đặc biệt của bài toán khi $P\equiv Q$: Cho tam giác $ABC$ với tâm đường tròn nội tiếp $I$. Gọi $H, K$ lần lược là trực tâm các tam giác $IAB, IAC$. Khi đó đường cao xuất phát từ đỉnh $A$ của tam giác $ABC$ chia đôi đoạn thẳng $HK$.

$$\begin{array}{| l | l |} \hline Ngockhanh99k48 & 3\\ \hline IHateMath & 1\\ \hline fatcat12345 & 3\\ \hline dogsteven & 4\\ \hline baopbc & 6\\ \hline QuangDuong12011998 & 2\\ \hline xuantrandong & 2\\ \hline mrjackass & 1\\ \hline vietnaminmyheart & 2\\ \hline BuiBaAnh & 1\\ \hline\end{array}$$




#644008 VMF's Marathon Hình học Olympic

Đã gửi bởi dogsteven on 07-07-2016 - 18:53 trong Hình học

$\boxed{\text{Bài toán 67}}$

Gọi $G$ là giao điểm của $AD$ với $(Q, QA)$ thì theo kết quả bài toán ??? ta suy ra $G$ thuộc $(DM)$

$AX$ là đường kính của $(Q, QA)$, khi đó cũng theo kết quả bài toán ??? ta có tam giác $MCX$ và $DLA$ đồng dạng.

Ngoài ra $TMX$ và $TDA$ đồng dạng nên $\dfrac{TM}{MC}=\dfrac{TM}{MX}:\dfrac{MC}{MX}=\dfrac{TD}{DA}:\dfrac{DL}{DA}=\dfrac{TD}{DL}$

Mà $LD\perp BC$ nên $\widehat{LDT}=\widehat{TMC}$

Do đó hai tam giác $TDL$ và $TMC$ đồng dạng, do đó $T\in (LC)$

Nhờ thầy và các bạn đề nghị bài toán tiếp theo.




#639005 VMF's Marathon Hình học Olympic

Đã gửi bởi dogsteven on 08-06-2016 - 20:58 trong Hình học

$\boxed{\text{Bài toán 25.}}$ Cho tam giác $ABC$. Đường tròn $(I)$ nội tiếp tam giác $ABC$ tiếp xúc với $BC, CA, AB$ tại $D, E, F$. Gọi $M, N$ là hai điểm trên $DF, DE$ sao cho $MN || EF$. $P$ là một điểm bất kỳ nằm trên $(I)$. $PM, PN$ cắt $(I)$ lần thứ $2$ tại $X, Y$. Gọi $Q$ là giao điểm của $BX$ và $CY$. Chứng minh rằng đường thẳng $AP$ và $AQ$ đối xứng với nhau qua $AI$




#644373 VMF's Marathon Hình học Olympic

Đã gửi bởi dogsteven on 10-07-2016 - 17:25 trong Hình học

Bài toán $77$ do Bảo đề nghị có lẽ được tạo ra nhờ bài toán sau: Cho tam giác $XYZ$ ngoại tiếp $(O)$. Các tiếp điểm của $(O)$ trên $YZ,ZX,XY$ là $A,B,C$. Một đường tròn tâm $O$ bất kỳ cắt tia đối của các tia $OA,OB,OC$(hoặc cắt các tia $OA,OB,OC$) lần lượt tại $L,M,N$.Khi đó $XL,YM,ZN$ đồng quy 

attachicon.gifUntitled.png

Bài này có lẽ đã cũ nhưng em(mình) chưa tìm được một lời giải nào hay, tính toán ít nên nhờ thầy và mọi người giúp đỡ :)

 

Em cũng muốn một lời giải đẹp cho bài toán này chứ cứ phải áp dụng cái định lý gì đó để chứng minh định lý Sondat thì vất vả lắm. Mong thầy và anh chị giúp ạ.




#639140 VMF's Marathon Hình học Olympic

Đã gửi bởi dogsteven on 09-06-2016 - 12:25 trong Hình học

$\boxed{\text{Bài toán 28.}}$

Bổ đề. Cho tứ giác $ABCD$ nội tiếp $(O)$ với $M$ là trung điểm $CD$ và $S$ là giao điểm của $AD$ và $BC$. Khi đó tiếp tuyến tại $A$ của $(SAB)$ tiếp xúc với $(O, OM)$

Gọi $I$ là tâm của $(SAB)$ và $T$ là giao hai tiếp tuyến tại $A$ và $B$ của $(SAB)$

Ta có $SI\perp CD$ nên $S(S, M, D, C)=-1$ nên $S, M, T$ thẳng hàng, mà $TA=TB, OA=OB, IA=IB$ nên $T, O, I$ thẳng hàng.

Mà $OM \perp CD$ nên $T$ là tâm vị tự ngoài của $(O, OM)$ và $(SAB)$

Do đó $TA, TB$ tiếp xúc với $(O, OM)$ nên ta có điều phải chứng minh.

 

Trở lại bài toán.

Gọi $X, Y, Z$ là tâm bàng tiếp các góc $A, B, C$ của tam giác $ABC$ và $T$ là trung điểm $YZ$

$K, L$ là giao điểm của $BC$ với $(XYZ)$ thì ta lại có cấu hình bài toán 17 tại đây

Chứng minh $X$ là tâm $KMNL$ có cách đơn giản hơn và đã được đăng ở trên group Bài toán hay - Lời giải đẹp - Đam mê toán học, các anh chị có thể lên tham khảo.

Tứ giác nội tiếp đường tròn tâm $X$ là $KLNM$ có $T$ là giao điểm của $KM, LN$ nên tiếp tuyến tại $M, N$ của $(TMN)$ tiếp xúc với đường tròn bàng tiếp góc $A$ tam giác $ABC$ và $(TMN)$ cũng chính là $(ABC)$ nên ta có điều phải chứng minh.

$$\begin{array}{| l | l |} \hline Ngockhanh99k48 & 4\\ \hline IHateMath & 1\\ \hline fatcat12345 & 4\\ \hline dogsteven & 5\\ \hline baopbc & 7\\ \hline QuangDuong12011998 & 2\\ \hline xuantrandong & 2\\ \hline mrjackass & 1\\ \hline vietnaminmyheart & 2\\ \hline BuiBaAnh & 1\\ \hline halloffame & 1\\ \hline\end{array}$$




#639142 VMF's Marathon Hình học Olympic

Đã gửi bởi dogsteven on 09-06-2016 - 13:09 trong Hình học

$\boxed{\text{Bài toán 29.}}$ Cho tam giác $ABC$ nội tiếp $(O)$ với đường đối trung $AD \;(D\in BC)$. Các điểm $E, F$ lần lược nằm trên $AC, AB$ sao cho $DE || AB$ và $DF || AC$. Gọi $A'$ là tâm đường tròn ngoại tiếp tứ giác $(BCEF)$. Xác định tương tự các điểm $B', C'$. Chứng minh rằng $AA', BB', CC'$ đồng quy.

Nguồn: Nhớ là trong một cuốn sách nào đó.




#690194 VMF's Marathon Hình học Olympic

Đã gửi bởi dogsteven on 11-08-2017 - 02:18 trong Hình học

Bài toán 199. Cho tam giác $ABC$ nội tiếp $(O)$, trực tâm $H$. $OH$ cắt $(O)$ tại $E, F$. $AH$ cắt $BC$ tại $D$. dựng các hình thang cân $ACBB'$ và $ABCC'$ với $BB' || AC, CC'|| AB$. $BC$ cắt $B'C'$ tại $X$. Chứng minh $E, F, X, D$ đồng viên.